Howdy, Stranger!

It looks like you're new here. If you want to get involved, click one of these buttons!

Modifying the wrong thing for PT64 S3 Q26

kkevinzzkkevinzz Core Member

I seem to be negating the wrong thing and was wondering if someone could clear up my confusion for PT64 S3 Q26. :(

The sentence that reads “It is rational not to acquire such information unless one expects that the benefit of doing so will outweigh the cost and difficulty of doing so”.

I negated “it is rational not to acquire” when doing the group 3 translation like below:

rational to acquire -> expects the benefit of doing so will

rather than:

/expect the benefit -> rational not to acquire

In this case, would the “not” be modifying when someone is being rational? I really need to review my grammar…. I seem to be super hung up when I see the word “not”

Thank you!!

Comments

  • KevinLuminateLSATKevinLuminateLSAT Alum Member
    983 karma

    I don't think your translation is incorrect. Those are just two different ways to express the same idea.

    "[It is rational not to acquire such information] unless [one expects that the benefit of doing so will outweigh the cost and difficulty of doing so.]"

    You take one of the bracketed ideas, negate, and put it on the left.

    IF it is NOT rational not to acquire such information --> One expects the benefit to outweigh the cost

    What does it mean for it to NOT be "rational not to acquire such info"? You could think of it as either "irrational not to acquire such info" or, by canceling out the double negative, "rational TO acquire such info". It looks like your translation used the latter.

    The other way to translate this statement:

    IF one does NOT expect the benefit to outweigh the cost --> It is rational not to acquire such info.

    In this case, the "not" in the necessary condition is applying to "acquiring such info". We're talking about the action of "not acquiring such info".  

    Can you let me know if there is something I'm missing about your question?

  • kkevinzzkkevinzz Core Member
    edited September 2023 25 karma

    Hello Kevin!

    Thank you so much for the reply and explanation. Your explanation clears up a lot of my confusion. Now I am a bit stumped on how to complete the entire setup in order to find the answer. We know that the conclusion says "Therefore, consumers who do not both to acquire such information are thereby behaving rationally.

    To set it up as I did initially for the first part and now adding my conclusion:

    rational to acquire information -> expects the benefits will outweigh


    behaving rationally if they expect benefits will outweigh -> bother to acquire information

    I understand group 4 negation but now I am a bit confused if I formulated my conclusion wrong and should instead say:

    bother to acquire -> /behaving rationally

    Thank you!

Sign In or Register to comment.